¿Prueba de que L=S=0L=S=0L=S=0 para subcapas de electrones llenas?

Se dice comúnmente que si tiene una subcapa llena, como pag 6 o d 10 que uno debe tener L = S = 0 Insinuando j = 0 y METRO j = 0 por lo que el átomo es esféricamente simétrico.

¿Por qué está claro que L = S = 0 ?

me queda claro que METRO L = METRO S = 0 . Esto se debe a que por cada electrón con metro yo o metro s hay otro electrón con metro yo y metro s tan claramente METRO L = i metro yo i = 0 y del mismo modo para S .

Pero en términos generales, si sumas varios giros y encuentras METRO L = METRO S = 0 esto NO es condición suficiente para tener L = S = 0 .

Esto es claro con el estado de triplete de dos spin-1/2 que es bien conocido por tener spin total S = 1 :

| ↑↓ + | ↓↑

Deseo una prueba de que para las conchas llenas obtenemos la propiedad anterior. Sé que la respuesta está relacionada con el requisito de antisimetrización para las funciones de onda de los electrones. Por ejemplo, la función de onda triplete anterior no es antisimétrica. Si tomara la combinación antisimétrica, tendría la camiseta que SÍ satisface S = 0 según sea necesario. Sin embargo, me gustaría una prueba de cómo el requisito de antisimetrización conduce a L = S = 0 por cualquier valor de L y 2 ( 2 L + 1 ) electrones

Hay una serie de preguntas similares en este sitio que puedo vincular si lo desea. Ninguno de ellos proporciona pruebas satisfactorias del tipo que deseo. A veces señalan que METRO L = METRO S = 0 Y dejar las cosas así. A veces señalan esto y agitan sus manos ante la anti-simetrización y lo dan por hecho. Me gustaría algo más convincente.

La razón por la que me preocupa toda esta situación es que parece que ciertos estados de momento angular son inaccesibles para fermiones y bosones de una manera que me parece más fuerte que lo que implica la exclusión de Pauli. Aunque tal vez subestime la exclusión de Pauli. Supongo que esto dice que si tengo, por ejemplo, 6 pag electrones, entonces solo hay UN estado que pueden ocupar. Pero la teoría general de la suma del momento angular dice que estos 6 giros 1 / 2 las partículas con momento angular orbital de espín-1 deberían tener como 36 estados de momento angular que deberían ocupar desde j = 0 a j = 9 .

Claramente no estoy pensando en esto correctamente. Agradecería ver la prueba que estoy buscando y aclarar mi intuición sobre la antisimetrización de Pauli.

editar: la respuesta a esta pregunta Dimensión del espacio de giro de Hilbert 1 / 2 partículas idénticas? aborda mi intuición sobre la exclusión de Pauli. La respuesta corta es que sí, una gran cantidad de estados de momento angular que estarían permitidos para partículas distinguibles simplemente se eliminan cuando se consideran los fermiones. Todavía busco una prueba convincente para arbitraria L , que, después de aplicar la antisimetrización, el único estado que queda en el S = L = j = 0 estado.

Cuando metro yo = metro s = 0 , ¿cómo elige el átomo el z -¿eje?
@JEB para estados generales de momento angular m = 0 no implica que el estado no tenga una orientación. Mira los armónicos esféricos Y 2 0 ( θ , ϕ ) . De alguna manera, la restricción adicional de la antisimetrización lo hace así. Este es el hecho por el cual me gustaría una prueba.
metro = 0 no tiene una orientación (una dirección vectorial), pero sí tiene una alineación tensorial con respecto a ± z ^ , Así que si metro = 0 ... ¿cómo sabría el átomo qué eje elegir?
@JEB Quizás llegó al estado en el que se encuentra debido a una interacción con un campo eléctrico polarizado.
un fondo mi rompe la simetría de modo que Y yo metro ya no son estados propios, así que no vayas allí . El punto que estaba diciendo es que si METRO = 0 para una capa llena, entonces eso es independiente de cómo elija el eje, y eso solo es cierto si L = 0 .
@JEB parece que estás insinuando que los átomos deben ser esféricamente simétricos en todas las circunstancias. Esto está mal. Consulte physics.stackexchange.com/questions/610064/… . Basta con mirar un orbital p (uno de estos tiene metro = 0 ) y verá que un átomo puede ser no esféricamente simétrico incluso si tiene metro = 0 . Es cierto que tiene que llegar por algo que rompa la simetría. Pero el elemento que rompe la simetría no necesita ser un campo de fondo constante. puede ser un pulso de luz que sale del
átomo en forma no esférica. Mi punto es que la forma puede ser no esférica incluso si metro = 0 . Ver el Y yo 0 que mencioné anteriormente. Sin embargo, por alguna razón, en el caso de anti-simetrización para fermiones, habiendo metro S = metro L = 0 Obliga al átomo a ser esféricamente simétrico, en contraste con el caso indistinguible. Estoy buscando la prueba de por qué este es el caso, especialmente con los fermiones.

Respuestas (3)

El estado general con metro = 0 es:

| L = yo , METRO = 0 = Y yo 0 ( θ , ϕ ) = 2 yo + 1 4 π PAG yo ( porque θ )

que describe una polarización tensorial con una alineación preferida a lo largo del eje que define θ . Si gira eso a diferentes ejes (cebado), entonces es una combinación lineal de armónicos esféricos con el mismo grado yo pero diferentes números cuánticos magnéticos:

Y yo 0 ( θ , ϕ ) = metro = yo yo C yo metro Y yo metro ( θ , ϕ )

que no es un estado con metro = 0 , a menos que yo = 0 .

En la pregunta afirmas que sabes que un átomo con una capa completa tiene METRO = 0 , pero no están seguros de L . El punto es que no hay un eje preferido, por lo que si cualquier eje que elija debe tener METRO = 0 , entonces L debe ser cero también.

Eso no está relacionado con el Principio de Exclusión de Pauli (PEP). El PEP resulta del teorema de la estadística de espín, que establece que la función de onda de fermiones idénticos debe ser antisimétrica bajo el intercambio de dos partículas cualesquiera:

Si:

ψ norte metro ( X 1 , X 2 ) = 1 2 [ ψ norte ( X 1 ) ψ metro ( X 2 ) ψ metro ( X 1 ) ψ norte ( X 2 ) ]

y

ψ norte metro ( X 2 , X 1 ) = 1 2 [ ψ norte ( X 2 ) ψ metro ( X 1 ) ψ metro ( X 2 ) ψ norte ( X 1 ) ] = ψ norte metro ( X 1 , X 2 )

dónde ( norte , metro ) etiquetar todos los números cuánticos que definen un estado, entonces si norte = metro :

ψ norte norte ( X 1 , X 2 ) = 0

que es el PEP. Tenga en cuenta que se cumple para todos los números cuánticos, no solo L = S = 0 .

Si está considerando seis electrones en un solo PAG -shell, entonces la función de onda se describe mediante un determinante de Slater:

ψ ( 1 , 2 , 3 , 4 , 5 , 6 ) = 1 6 ! ×
| Y 1 1 ( θ 1 , ϕ 1 ) | 1 Y 1 1 ( θ 2 , ϕ 2 ) | 2 Y 1 1 ( θ 3 , ϕ 3 ) | 3 Y 1 1 ( θ 4 , ϕ 4 ) | 4 Y 1 1 ( θ 5 , ϕ 5 ) | 5 Y 1 1 ( θ 6 , ϕ 6 ) | 6 Y 1 0 ( θ 1 , ϕ 1 ) | 1 Y 1 0 ( θ 2 , ϕ 2 ) | 2 Y 1 0 ( θ 3 , ϕ 3 ) | 3 Y 1 0 ( θ 4 , ϕ 4 ) | 4 Y 1 0 ( θ 5 , ϕ 5 ) | 5 Y 1 0 ( θ 6 , ϕ 6 ) | 6 Y 1 1 ( θ 1 , ϕ 1 ) | 1 Y 1 1 ( θ 2 , ϕ 2 ) | 2 Y 1 1 ( θ 3 , ϕ 3 ) | 3 Y 1 1 ( θ 4 , ϕ 4 ) | 4 Y 1 1 ( θ 5 , ϕ 5 ) | 5 Y 1 1 ( θ 6 , ϕ 6 ) | 6 Y 1 1 ( θ 1 , ϕ 1 ) | 1 Y 1 1 ( θ 2 , ϕ 2 ) | 2 Y 1 1 ( θ 3 , ϕ 3 ) | 3 Y 1 1 ( θ 4 , ϕ 4 ) | 4 Y 1 1 ( θ 5 , ϕ 5 ) | 5 Y 1 1 ( θ 6 , ϕ 6 ) | 6 Y 1 0 ( θ 1 , ϕ 1 ) | 1 Y 1 0 ( θ 2 , ϕ 2 ) | 2 Y 1 0 ( θ 3 , ϕ 3 ) | 3 Y 1 0 ( θ 4 , ϕ 4 ) | 4 Y 1 0 ( θ 5 , ϕ 5 ) | 5 Y 1 0 ( θ 6 , ϕ 6 ) | 6 Y 1 1 ( θ 1 , ϕ 1 ) | 1 Y 1 1 ( θ 2 , ϕ 2 ) | 2 Y 1 1 ( θ 3 , ϕ 3 ) | 3 Y 1 1 ( θ 4 , ϕ 4 ) | 4 Y 1 1 ( θ 5 , ϕ 5 ) | 5 Y 1 1 ( θ 6 , ϕ 6 ) | 6 |

A partir de eso, puede calcular la densidad de probabilidad frente a la coordenada angular, y se verá así:

PAG ( θ , ϕ ) | Y 1 1 θ , ϕ | 2 + | Y 1 0 θ , ϕ | 2 + | Y 1 1 θ , ϕ | 2
| pecado θ mi + i ϕ | 2 + | 2 porque θ | 2 + | pecado θ mi ϕ | 2 = 2 ( pecado 2 θ + porque 2 θ ) = 2

es decir, es esféricamente simétrica. Simetría esférica significa L = 0 .

Esto es válido para cualquier orden:

metro = yo yo | Y yo metro ( θ , ϕ ) | 2

no depende de θ ni ϕ . Por lo tanto, las capas llenas siempre son esféricamente simétricas con un momento angular total L = 0 .

Sin embargo, existe una conexión profunda entre la antisimetría y la invariancia rotacional. Por ejemplo: el símbolo antisimétrico Levi Civitta, ϵ i j k , es un tensor isotrópico. Esto surge a través de la dualidad de Schur-Weyl, que describe los subespacios de tensores cerrados rotacionalmente a través de las representaciones del grupo de permutación y los diagramas de Young. Las dimensiones de los subespacios se pueden calcular con la fórmula de longitud de gancho "la notable".

La permutación antisimétrica corresponde a un subespacio de dimensión 1, que es un escalar (por lo tanto, esféricamente simétrico). El ejemplo más simple se incluye en su pregunta, donde combinó dos representaciones 2D (espinores) y obtuvo un triplete simétrico y un singlete antisimétrico:

2 2 = 3 S 1 A

El triplete se transforma como un vector y el singlete como un escalar.

Del mismo modo, si combina 3 vectores (por ejemplo, 3 orbitales P), obtiene:

3 3 3 = 10 S 8 METRO 8 METRO 1 A

donde el simétrico 10 es L = 3 y L = 1 , los octetos son L = 2 y L = 1 , y el singlete completamente antisimétrico es L = 0 . Puede verificar esto trabajando laboriosamente a mano con los coeficientes de Clebsch Gordon.

Tengo algunas conclusiones/preguntas. Primero, muestra explícitamente que el estado de 6 electrones que llena el pag -la concha es isotrópica. A continuación, afirma que esto es válido para cualquier valor de L . Tengo curiosidad por escuchar un desarrollo de la prueba de esta última afirmación, esa es esencialmente toda mi pregunta. Las próximas secciones sobre la relación entre la antisimetría y la invariancia rotacional parecen ser la clave. Especialmente que la permutación antisimétrica es de dimensión uno. Pero claramente no todos los estados antisimétricos son esféricamente simétricos. Si solo tengo dos electrones en pag -orbitales...
Todavía forman un estado determinante de pizarra antisimétrico, pero no es rotacionalmente simétrico. Entonces es algo como, hay 3 estados orbitales disponibles y 2 estados de espín disponibles. Entonces, el espacio de Hilbert de una sola partícula tiene 6 dimensiones. Si ahora hace 6 copias de ese espacio (6 electrones) pero se restringe a permutaciones antisimétricas, está diciendo que tiene que ser unidimensional. ¿Se puede probar eso fácilmente?
Supongo que la prueba es algo así como, dado que es antisimétrica, sabes que tienes que tener una copia de cada estado y solo hay una forma de elegir 6 objetos únicos de un grupo de 6 objetos.
¿Cómo pudiste asignar las etiquetas S, M y A a los diversos subespacios al final de la respuesta? También pregunté a physics.stackexchange.com/questions/631984/… que puede llegar más directamente a lo que estoy tratando de averiguar. Si puede, tal vez podría responder allí para aclarar si tiene más sentido.
Obtiene las simetrías de algo llamado "The Young Symmetrizer", por lo que para cada cuadro estándar tiene una receta para obtener las permutaciones rotacionalmente cerradas.
¿Podría decir algo más sobre cómo se usa el Young Symmetrizer para determinar las simetrías de los estados de momento angular compuestos? He tratado de mirar dentro de mí mismo, pero parece un poco complicado. Agradecería una respuesta aquí, o en la otra pregunta que vinculé en mi otro comentario. ¡También se agradecería una buena referencia si eso es más fácil de proporcionar!

Me gustaría compartir una respuesta, basada en la prueba (boceto) que encontré en L. Marchildon: Quantum Mechanics. From Basic Principles to Numerical Methods and Applications, capítulo 18 (p. 403f.) .

Dado que solo nos interesan las subcapas llenas, primero definamos el giro total y el operador orbital angular para nuestra subcapa de interés,

L := i subcapa L i , S := i subcapa S i .
El índice i recorre todos los electrones de esa subcapa.

Con respecto a estos operadores 'totales', también definimos operadores de subida y bajada,

L := L X i L y S := S X i S y .
Los operadores de subida se definen como el conjugado hermitiano de los operadores de bajada, es decir ( L ) = L + = L X + i L y y de manera similar, ( S ) = S + = S X + i S y .

El efecto del operador de descenso. L en un estado cuántico Ψ con números cuánticos METRO L y L es bajar METRO L por 1, pero mantener L fijo mientras METRO L > L . Sin embargo, por un mínimo METRO L ( METRO L = L ) el operador de descenso destruye el estado Ψ . Análogamente, podemos describir el efecto del operador de elevación L + : Si METRO L < L , actúa sobre Ψ elevando METRO L por 1 y manteniendo L fijado. Si METRO L es máximo, es decir METRO L = L , destruye el estado Ψ . (Lo mismo se aplica a los operadores de reducción y elevación de espín con respecto a los números cuánticos METRO S y S .)
Hasta ahora, esto no es nada nuevo.


Observación

Permítanme comentar aquí que, en general, los operadores de elevación y disminución de un solo electrón L i ± no conmutan con permutaciones de partículas, pero sí lo hacen los operadores de subida y bajada total (por ejemplo , para una subcapa). Esto significa que, después de actuar con L ± = i L i ± en un estado completamente (anti-)simetrizado permanecerá (anti-)simétrico. Sin embargo, la acción de un solo operador L i ± destruirá (en general) cualquier (anti-)simetría.

Por ejemplo, considere dos electrones (con coordenadas r 1 y r 2 ) en los Estados Y yo = 1 metro = 1 | y Y yo = 1 metro = 0 | (Por supuesto, esto es físicamente muy improbable; solo tiene un propósito de demostración aquí). Un estado de dos electrones completamente anti-simetrizado es el determinante de Slater

Ψ ( r 1 , r 2 ) = Y yo = 1 metro = 1 ( r 1 ) Y yo = 1 metro = 0 ( r 2 ) | 1 2 Y yo = 1 metro = 0 ( r 1 ) Y yo = 1 metro = 1 ( r 2 ) | 1 2 .
Ahora, actuando con L en Ψ ( r 1 , r 2 ) encontramos,
L Ψ ( r 1 , r 2 ) = ( L 1 + L 2 ) Ψ ( r 1 , r 2 ) = 2 Y yo = 1 metro = 0 ( r 1 ) Y yo = 1 metro = 0 ( r 2 ) | 1 2 2 Y yo = 1 metro = 1 ( r 1 ) Y yo = 1 metro = 1 ( r 2 ) | 1 2 + 2 Y yo = 1 metro = 1 ( r 1 ) Y yo = 1 metro = 1 ( r 2 ) | 1 2 2 Y yo = 1 metro = 0 ( r 1 ) Y yo = 1 metro = 0 ( r 2 ) | 1 2 .
(La aparición del factor 2 se puede ver desde la identidad L L + = L 2 L z 2 L z .) Claramente, este estado es nuevamente antisimétrico, aunque no es un determinante de Slater.

Se puede verificar que un estado antisimétrico general, que puede escribirse como una suma de determinantes de Slater, permanece antisimétrico después de la acción de L ± . (Si cree en esta afirmación para cualquier determinante de Slater, entonces simplemente use que la permutación de partículas es una operación lineal).


Ahora, considere un estado Ψ con valores propios L , METRO L , S , METRO S con respecto a los operadores de subcapa total que definimos anteriormente. Además, Ψ debe describir una subcapa llena . A continuación probaremos que METRO L = L = 0 y METRO S = S = 0 similar.

Simplifiquemos un poco la situación y comencemos observando el momento angular. L y METRO L y considere lo que sucede cuando actuamos con L en Ψ . por definición de L , la acción del operador de reducción de la subcapa es solo la suma de los operadores de reducción L i por cada electrón i ,

L = i subcapa L i .
(Como hemos visto, tomar la suma es importante para mantener el resultado antisimetrizado).

Desde Ψ es un estado completamente anti-simetrizado, puede contener varios términos/sumandos. Usamos una base propia de la L i (o mejor L i 2 y L i z ) como base de una partícula con respecto a la cual antisimetrizamos; vea el ejemplo en el comentario anterior. Esto significa que, en cada sumando, el operador L i actúa sobre el estado del electrón i con algún número cuántico bien definido metro yo i .

  • Si metro yo i > yo i , entonces L i simplemente baja metro yo i metro yo i 1 . Pero debido a que la subcapa está completamente ocupada, otro electrón ya ocupa este estado. Entonces tenemos dos electrones en el mismo estado. Actuando con el operador de reducción total de la subcapa L en Ψ asegura la antisimetría del estado final total L Ψ , como hemos visto en el comentario anterior. Antisimetrizar un estado con dos electrones que ocupan el mismo estado produce cero (principio de exclusión de Pauli).
  • Si metro yo i = yo i , es decir metro yo i es mínimo, no podemos bajar el número cuántico metro yo i más allá y este término/suma de Ψ Esta destruido.

En total, encontramos L Ψ = 0 .

Por un razonamiento análogo, usando las propiedades del estado completo de la subcapa Ψ al actuar con L + -- podemos deducir L + Ψ = 0 .

Ahora viene el truco (puede que ya adivines lo que viene a continuación): dado que los operadores de subida y bajada destruyen Ψ , su número cuántico METRO L debe ser mínimo y máximo simultáneamente . Esto solo funciona si L = 0 que restringe METRO L ser cero también.

Al principio también he definido un operador de subida y bajada S ± para el giro. Aunque no doy los detalles aquí, la prueba para S = METRO S = 0 funciona igual que para el momento angular que hemos hecho aquí.

Otro enfoque

(Relacionado con la respuesta de JEB)

Si crees que las cáscaras vacías tienen L = S = 0 , entonces puede convencerse de que las capas llenas tienen exactamente la misma propiedad mediante una transformación de partículas-agujeros. Básicamente, esto equivale a asignar la creación de electrones a los operadores de aniquilación de huecos (y la aniquilación de electrones a los operadores de creación de huecos), mientras que también se asigna al hueco correspondiente el momento angular magnético inverso y el número cuántico de espín que el electrón, es decir yo yo , metro yo metro yo , s s , metro s metro s . Esta asignación asegura que la representación (matricial) de los operadores L y S no son cambiados por la transformación, de modo que todas las propiedades con respecto a L y S la aplicación a carcasas electrónicas vacías también se aplica a las carcasas con orificios llenos. (Los detalles de esta transformación también se pueden encontrar en los libros de texto de física atómica y en las notas de conferencias en línea).

Por supuesto, esto está relacionado con el hecho elaborado por JEB: cualquier objeto, que es invariante bajo rotaciones, es decir, acciones bajo grupo generado por L , se transforma como tensor de rango cero, es decir, como una cantidad escalar en nuestro ejemplo de momento angular y rotaciones espaciales. De hecho, esto es independiente del principio de exclusión de Pauli y del espacio sobre el que actúa la representación del grupo.

CONTEXTO

La pregunta aquí pide una prueba de que para una subcapa llena el momento angular total, L , y el momento angular de espín total, S , son tales que

L = S = 0 .

Arriba, @JEB proporciona una respuesta a la pregunta. No tengo problemas con esa respuesta, excepto que no se menciona (hasta donde puedo decir) de S . Por lo tanto, por lo que puedo decir, la pregunta original sigue sin respuesta. en mi respuesta no aporto prueba , que es lo que se pide; más bien, proporciono una demostración que indica mi racional. Me imagino que tal racional podría extenderse a una prueba.

DEMOSTRACIÓN

Magnesio ( Z = 12 ) es un metal divalente con configuración electrónica de estado fundamental [Ne]3s 2 . ¿Cuál es el momento angular del estado fundamental?

En el esquema LS ( https://en.wikipedia.org/wiki/Angular_momentum_coupling ), las reglas son que

L [ norte mi ] 3 s 2 = 1 + 2   y S [ norte mi ] 3 s 2 = s 1 + s 2 .

para los 1s 2 ,2 s 2 , 2p 6 ,3s 2 estado fundamental, tenemos dos electrones de valencia en el s caparazón. Todos y cada uno de los electrones del s shell tiene el número cuántico = 0 . Por lo tanto 1 = 2 = 0 . Así, los posibles valores de los números cuánticos para

L [ norte mi ] 3 s 2 = 1 + 2   con   1 = 0 y 2 = 0 ,
son
L [ norte mi ] 3 s 2 = { ( 0 + 0 ) , , ( | 0 0 | ) } = { 0 } .
Por eso,
| L [ norte mi ] 3 s 2 | = 0 ( 1 + 0 ) = 0 ,
y
L [ norte mi ] 3 s 2 = 0 .

para los 1s 2 ,2 s 2 , 2p 6 ,3s 2 estado fundamental, tenemos dos electrones de valencia en el s caparazón. Todos y cada uno de los electrones tienen el número cuántico s = 1 / 2 . Por lo tanto s 1 = s 2 = 1 / 2 . Así, los posibles valores de los números cuánticos para

S [ norte mi ] 3 s 2 = s 1 + s 2   con   s 1 = 1 / 2 y s 2 = 1 / 2 ,
parece ser
S [ norte mi ] 3 s 2 = { ( 1 / 2 + 1 / 2 ) , , ( | 1 / 2 1 / 2 | ) } = { 1 , 0 } .
Además parece que
| S [ norte mi ] 3 s 2 | = { 0 ( 1 + 0 ) , 1 ( 1 + 1 ) } .
Dicho esto, debemos tener en cuenta lo siguiente.

“En el caso especial de las configuraciones terrestres de electrones equivalentes, el espín y el momento angular orbital del término de energía más baja siguen algunas reglas emipircales llamadas reglas de Hund: el término de energía más baja tiene el mayor valor de S consistente con el principio de exclusión de Pauli. (Física Atómica, Pie, 2005, p.81)''

En este ejemplo, estamos tratando claramente con la configuración de tierra y el término de energía más baja. Según el principio de exclusión de Pauli no podemos tener dos electrones con el estado cuántico

| norte , , metro , s , metro s >= | 3 , 0 , 0 , 1 / 2 , + 1 / 2 > ;
del mismo modo, no podemos tener dos electrones con el estado cuántico
| norte , , metro , s , metro s >= | 3 , 0 , 0 , 1 / 2 , 1 / 2 > .
Por lo tanto, para la configuración de tierra, uno de cada uno de los electrones (pero no ambos) debe cuantificarse de la siguiente manera:
| ψ 1 >= | norte , , metro , s , metro s >= | 3 , 0 , 0 , 1 / 2 , + 1 / 2 > | ψ 2 >= | norte , , metro , s , metro s >= | 3 , 0 , 0 , 1 / 2 , 1 / 2 >
El valor de S = 1 , lo que conduciría a un valor distinto de cero | S [ norte mi ] 3 s 2 | , no es físicamente realizable ya que para el espín necesitaríamos dos electrones cuyos momentos angulares de espín individuales no sean iguales y opuestos. Esto sería inconsistente con el principio de exclusión de Pauli (es decir, los dos momentos angulares de espín deben ser iguales y opuestos y, por lo tanto, no dar como resultado un momento angular de espín neto). De acuerdo con la regla de Hund, encontramos que
S [ norte mi ] 3 s 2 = { 0 } .
Por eso,
| S [ norte mi ] 3 s 2 | = 0 ( 1 + 0 ) = 0 ,
y
S [ norte mi ] 3 s 2 = 0 .

En resumen, he demostrado un caso particular tal que para una subcapa llena el momento angular total, L , y el momento angular de espín total, S , son idénticamente cero. Es decir, encuentro

L [ norte mi ] 3 s 2 = S [ norte mi ] 3 s 2 = 0 .